How many solutions does the system have

How Many Solutions Does The System Have

Answers

Answer 1

Answer:

Step-by-step explanation:

28x-12y=16 and -6x-12y=3

or

14x-6y=4 and -14-28y=1

or

-28x-12y=16 and 28x-56y=14


Related Questions

The lateral area of a right cone which has a base diameter of 4 units and a height of 10 units is:

Answers

Answer:

≈64.08

Step-by-step explanation:

Lateral Area=πr[tex]\sqrt{h^{2} +r^{2} }[/tex]

Radius is 1/2 of the diameter so r=2

Height=10

Substitute:

: [tex]3.141(2)\sqrt{10^{2}+2^{2} } \\6.282\sqrt{100+4} \\6.282\sqrt{104} \\ 64.066.282*10.2\\ 64.076\\\\[/tex]

64.076 rounded = 64.08

What is the slope of the graph? slope = -1/3 slope = -3 slope = 3 slope = 1/3

Answers

Answer:

The slope is -3

Step-by-step explanation:

This is so because the line in pointing down, leading to the fact that the slope is negative, and rise over run is 3/1......

Therefore, the slope of the line is -3

To find the slope of the line, first start with a point on the graph.

Let's use the point (0,3) which we call point A.

The other point we will call point B.

Now remember that slope can be found using the ratio rise/run

between any two points that are on that line.

To get from point A to point B along this line, we must

first go down 3 units so we say that our rise is -3.

From there we move 1 unit to the right so our run is 1.

So our slope or rise/run is -3/1 which reduces to -3.

You are going to use an incline plane to lift a heavy object to the top of a shelving unit with a height of 5 ft. The base of the incline plane is 15 ft from the shelving unit. What is the length of the incline​ plane?

Answers

Answer:

15.8 ft

Step-by-step explanation:

The inclined plane, the base of and the shelving unit form the shape of a right angled triangle.

The hypotenuse is the length of the inclined plane, h.

The base of the triangle is 15 ft.

The height of the triangle is 5 ft.

To find the hypotenuse, h, we have to use Pythagoras rule:

[tex]h^2 = a^2 + b^2[/tex]

where a = height of the triangle

b = base of the triangle

Therefore:

[tex]h^2 = 5^2 + 15^2\\\\h^2 = 25 + 225 = 250\\\\h = \sqrt{250}\\ \\h = 15.8 ft[/tex]

The inclined plane is 15.8 ft long.

Brass contains copper and zinc in the ratio 2:1. How much ZINC is there in 750 g of brass?

Answers

Answer:

250

Step-by-step explanation:

copper: zinc : total

2              1      2+1 =3

We have 750 brass

750/3 = 250

Multiply each by 250

copper: zinc : total

2*250     250  3*250

500        250       750


Before graduating this year, a senior homeroom was given a survey. Of those surveyed, 24% felt they learned better at home. Of this group, 80% said they plan on taking an online course in college. Of the students who felt they did not learn better at home, 40% said they plan on taking an online course in college

Part A
What is the probability a person who does not plan on taking an online course felt they learned better at home?

A : 2/21
B : 24/125
C : 38/125
D : 19/31
E :None of these

Part B
What is the probability a person who does plan on taking an online course felt they did not learn better at home?

A : 2/21
B : 24/125
C : 38/125
D : 19/31
E : None of these

Answers

Answer:

(A) The correct option is (A).

(B) The correct option is (E).

Step-by-step explanation:

The events can be defined as follows:

X =  students felt they learned better at home

Y = students plan on taking an online course in college

The information provided is:

P (X) = 0.24

P (Y|X) = 0.80

P (Y|X') = 0.40

[tex]P(Y'|X)=1-P(Y|X)\\=1-0.80\\=0.20[/tex]

[tex]P(Y'|X')=1-P(Y|X')\\=1-0.40\\=0.60[/tex]

The Bayes' theorem states that the conditional probability of an event E[tex]_{i}[/tex] given that another event A has already occurred is:

[tex]P(E_{i}|A)=\frac{P(A|E_{i})P(E_{i})}{\sum {P(A|E_{i})P(E_{i})}}[/tex]

(A)

Compute the probability a person who does not plan on taking an online course felt they learned better at home as follows:

Use the Bayes' theorem.

[tex]P(X|Y')=\frac{P(Y'|X)P(X)}{P(Y'|X)P(X)+P(Y'|X')P(X')}[/tex]

              [tex]=\frac{0.20\times 0.24}{(0.20\times 0.24)+(0.60\times 0.76)}\\\\=0.09524\\\\\approx 0.095[/tex]

Thus, the probability a person who does not plan on taking an online course felt they learned better at home is 0.095 or 2/21.

(B)

Compute the probability a person who does plan on taking an online course felt they did not learn better at home as follows:

[tex]P(X'|Y')=1-P(X|Y')\\=1-0.095\\=0.905[/tex]

Thus, the probability a person who does plan on taking an online course felt they did not learn better at home is 0.905.

pleass more math help

Answers

Can’t answer this question

k (t) = 10t - 19
k(-7) =

Answers

Answer:

k(-7) = - 89

Step-by-step explanation:

k (t) = 10t - 19

Let t = -7

k(-7) =10 * -7 - 19

        =-70-19

        -89

Answer:

[tex]k (-7) = -89[/tex]

Step-by-step explanation:

[tex]k (t) = 10t - 19[/tex]

[tex]k (-7) = 10(-7) - 19[/tex]

[tex]k (-7) = -70 - 19[/tex]

[tex]k (-7) = -89[/tex]

The line plot shows the result of a survey asking students how many hours they spent reading last week. How many students spent 5 or more hours reading?

Answers

Answer:

15 total students

Step-by-step explanation:

Based on the line plot picture that is attached below it can be calculated that a total of 15 students spent 5 or more hours last week reading. This can be easily calculated by adding all the x's on the marks of 5 hours or more. With both the boys and girl students combined the tally was the following

5 hours : 4 students

6 hours : 1 student

7 hours : 3 students

8 hours : 3 students

9 hours : 0 students

10 hours: 2 students

11 hours : 2 students

12 hours : 0 students

adding up to 15 total students

En un programa de televisión se hacen 30 preguntas. Por cada respuesta correcta se suman 8 puntos, por cada respuesta errónea se restan 5 puntos y por aquellas preguntas que no se contesten no se suman ni se restan puntos. Si un participantes obtuvo 13 puntos, ¿Cuantas respuestas erróneas pudo tener?

Answers

Answer:

7 wrong answers.

Step-by-step explanation:

We know that when you answer a question well, you earn 8 points and a bad one loses 5 points, this means that when you answer 1 good and 1 bad, there is a total of 3 points (8-5).

This means that when answering 5 good and 5 bad, in total it would be 3 * 5 points, that is to say 15 points, that is, it goes through 2 points. To subtract 2 points, you would have to answer 1 good and 2 bad, (8 - 2 * 5), which turns out to be -2 points.

In total there would be 6 questions right and 7 questions wrong, like this:

8 * 6 + 5 * 7 = 13

13 points, in 13 questions (6 correct + 7 incorrect)

The rest of the questions were not answered so that the score does not go up or down.

Which means you got 7 wrong answers.

Solve for m.

3>m+8/5
plz helppppp

Answers

Answer:

m < 1.4

Step-by-step explanation:

Make [tex]\frac{8}{5}[/tex] a decimal.

1.6

3 > m + 1.6

m < 1.4

John has two jobs and earns a total of $2,345 per month. What percent of his gross income does John receive from his second job where he makes $609.70 a month?

Answers

Answer:

10000000.9

Step-by-step explanation:

please solve y = 3x - 1​

Answers

Step-by-step explanation:

i think question is not complete.

8. Where will the hour hand of a clock stop if it starts:
a.
from 7 and turns through 1 right angle?
b. from 11 and turns through 3 right angles

can you plz say me the answer​

Answers

Answer:

a. 11

b. 9

Step-by-step explanation:

thats the answer

Please help me!!!!!!

Answers

Answer:

C

Step-by-step explanation:

The addition of the x^5 term makes it non proportional

Answer:

C.

Step-by-step explanation:

A. y = (6x + 3) - 3= 6x

B. y = - 15x

D. = - 1/3 x

All of these are proportional because they have general formula y = kx.

Identify the like terms in the expression.

Answers

Answer:D

The answer is D.

Step-by-step explanation:

Like terms are only related to the variable at the end, and the variable also has to have the same exponent or it is not a like term.

PLS HELP

Simplify the function f(x) =
3x
(81)
4
Then determine the key aspects of the function.
The initial value is
The simplified base is
The domain is
The range is

Answers

Answer:

Step-by-step explanation:

This follows the form

[tex]y=a(b)^x[/tex]

where a is the initial value and b is the base with the exponent.  Using that information, we can see that the initial value in our function is 1/3. Simplifying the base will take some work. Let's first rewrite this is a radical:

[tex]81^{\frac{3x}{4} }=\sqrt[4]{81^{3x} }[/tex]

Now let's break up 81 into its factors. 81 is 9*9 which is 3*3*3*3.  Therefore,

[tex]81=3^4[/tex]

We will use that as a simplification:

[tex]\sqrt[4]{(3^4)^{3x}}[/tex]  which simplifies to

[tex]\sqrt[4]{3^{12x}}[/tex]

Rewriting that as an exponent looks like this:

[tex]3^{\frac{12x}{4}}[/tex] which simplifies to

[tex]3^{3x}[/tex]

That's the answer for the second part.  The whole exponential equation now is

[tex]f(x)=\frac{1}{3}(3)^{3x}[/tex]

The domain for an exponential function is all real numbers and the range is

y > 0

Answer:

1/3

27

all real numbers

y > 0

ON EDGE

Step-by-step explanation:

The circumference of the earth is given. 


Circumference of earth: 24,901 miles


What is the diameter of earth? Round your answer to the nearest tenth. Use 3.14 for π.


Answers

Answer:

7930.3 miles = d

Step-by-step explanation:

The circumference equals

C = pi *d

24901 = 3.14 d

Divide each side by 3.14

24901 / 3.14 = d

7930.254777 = d

Rounding to the nearest tenth

7930.3 =d

Jasper decided to save $100 at the end of each month for a year and deposit it in a bank account that earns an annual interest rate of 0.3%, compounded monthly. Use the formula for an annuity, F, to determine how much money will be in the account at the end of the 6th month, rounding your answer to the nearest penny.
Note: Your interest rate must be converted to a decimal

Answers

Answer: 600.38

Step-by-step explanation:

just put in the answer you lazy

Please answer correctly !!!! Will mark brainliest !!!!!!!!!!!!

Answers

Answer:

[tex]x^2+10x+24[/tex]

Step-by-step explanation:

So just multiply the sides:

[tex]x^2+6x+4x+24[/tex]

Which is:

[tex]x^2+10x+24[/tex]

What is the shape of the cross-section formed when a cylinder intersects a
plane as shown in the drawing?

Answers

Answer:

circle,

Step-by-step explanation:

disc in the middle vertically in this case is equal to the bases

The intersection is called an Oval. Hence the correct option is A. An oval in mathematics is a shape just like a circle but with an elongated outline like the shape of an egg.

What is a Cross-section?

A cross-section is a surface, an area that is created or exposed by executing a straight cut across or through a shape.

Cross-sections in technical drawings are used for depicting the internal view of an object that is three-dimensional.

Learn more about Cross-Sections are:

https://brainly.com/question/10511133

(3x5−2x4−5)−(2x4+x2−10) Subtract the two polynomials

Answers

Answer:

3x^5-4x^4-x^2+5

Step-by-step explanation:

(3x^5−2x^4−5)−(2x^4+x^2−10)

Distribute the minus sign

(3x^5−2x^4−5)−2x^4-x^2+10

Combine like terms

3x^5-4x^4-x^2+5

Hello!

Answer:

[tex]\boxed{ \bf 3x^5~-~4x^4~-~x^2~+~5}[/tex]

__________________________________Explanation:

(3[tex]x^{5}[/tex] - 2[tex]x^{4}[/tex] - 5) - (2[tex]x^{4}[/tex] + x² - 10)

Drop the brackets:

3[tex]x^{5}[/tex] - 2[tex]x^{4}[/tex] - 5 - 2[tex]x^{4}[/tex] - x² + 10

Combine Like Terms:

3[tex]x^{5}[/tex] - 2[tex]x^{4}[/tex] - 2[tex]x^{4}[/tex] - x² - 5 + 10

3[tex]x^{5}[/tex] - 4[tex]x^{4}[/tex] - x² + 5

Which angle in ADEF has the largest measure?

Answers

Answer:

F is the largest angle

Step-by-step explanation:

The largest angle is opposite the largest side.  The smallest angle is opposite the smallest side.

The largest side is 4 so the largest angle is F

How many digits will be in the quotient?
39 4,641

Answers

Answer:

the answer is 119 so three digits

Step-by-step explanation:

Answer:

3 digits

Step-by-step explanation:

4641/39=119

A man divides his 360 cattle between his son in the ratio 7: 6: 5. Find the smallest share​

Answers

360/18=20

20*7=140

20*6=120

20*5=100

Answer=140:120:100

Answer: smallest share is 100

Step-by-step explanation:

7:6:5

Add the numbers so 7+6+5=18

Divide 360 by 18 which equals 20

Multiply each ratio by 20 so...

7×20:6×20:5×20

140:120:100

What the answer to this

Answers

Answer:

I think it might be a I don't know for sure though I just need some more points so I can ask a question myself sorry if this didn't help

The price of a computer was decreased by 7% to £500. What was the price before the decrease? Give your answer to the nearest penny.

Answers

Answer:

£537.63

Step-by-step explanation:

Let the original price of the computer be x.

The price of a computer was decreased by 7% to £500. This implies that:

x - 7/100 * x = 500

x - 0.07x = 500

0.93x = 500

x = 500/0.93 = £537.63

The price before the decrease was £537.63.

Can someone please help

Answers

Answer:

The numbers to input are 2, 2 , 7

Step-by-step explanation:

252 = 2² * 3² * 7

So the numbers to input are 2, 2 , 7

Find the value of a . A.18 B.21 C.20 D.17

Answers

Answer:

a =18

Step-by-step explanation:

The two angles are vertical angles and vertical angles are equal

6a +11 = 2a+83

Subtract 2a from each side

6a-2a +11 = 2a-2a+83

4a +11 =83

Subtract 11 from each side

4a +11 -11 = 83-11

4a = 72

divide each side by 4

4a/4 = 72/4

a =18

(6) Work out
5 1
6 12

Answers

Answer:

=123

Step-by-step explanation:

HELPPPPPP ITS ABOUT EQUATIONSSSS HELPPPPP Explanation needed HELPPPPP PLEASEE THIS IS THE LAST QUESTION LEFTTTTTTT

Answers

Answer:

C. the average total cost for the first month of a gym membership

Step-by-step explanation:

x=1 is for month 1,

the value of y includes 1-month fee and one off payment, so this is the average total for the first month of membership

Answer:

It is the average total cost for the first month of a gym membership

Step-by-step explanation:

y = 34.99x+49

The 49 is the cost to join the gym and the 34.99 is the monthly cost

Let x =1 which is the cost after one month

It includes the cost to join and the 1st month membership

It is the average total cost for the first month of a gym membership

Other Questions
PLZ HELP I CAN'T FAIL THIS LESSON ILL GIVE YOU 20 POINTS!!!!How do the Woodcutter's Wife's actions in Act 1 help develop the theme in Act 2?READ THE STORY BELOW >>>>>>>>adapted from The Emperor's Testby Augusta StevensonCHARACTERSTHE EMPERORTHE GENERALTHE CAPTAINTHE AIDETHE MAYOR'S WIFE AND HER SONTHE MERCHANT'S WIFE AND HER SONTHE WOODCUTTER'S WIFE AND HER SON, PIERREACT I, SCENE I: Inside THE EMPEROR'S tent.AIDE:(Speaking to those outside the tent) You will please enter.[Enter the MAYOR'S WIFE and SON; the MERCHANT'S WIFE and SON.]GENERAL:His Majesty wishes you to leave your sons here in camp until evening.MAYOR'S WIFE:General, could you not tell us the Emperor's plans?GENERAL:The Emperor wishes a guide who can lead him safely through the forest. The lad chosen will be made an aide.MAYOR'S WIFE:I thought only princes were chosen for the Emperor's aides.GENERAL:They have always been princes. This is a great opportunity for the lads of this village. A test will be given every boy who comes.[Enter the WOODCUTTER'S WIFE and SON.]WOODCUTTER'S WIFE:I heard the Emperor wanted a guide.MAYOR'S WIFE:The Emperor only wants the boys of the best families.WOODCUTTER'S WIFE:(Sighing) Pierre is a smart boy. If the Emperor could only see himMERCHANT'S WIFE:(Interrupting) The Emperor wants a boy with proud manners such as our boys have.EMPEROR:(Entering) Fiddlesticks!THE LADIES:(Bowing) Your Highness!WOODCUTTER'S WIFE:I am sorry, your Majesty. I didn't know how it was. Come, Pierre.[She turns to go.]EMPEROR:Remain. Pierre shall have the test with the others. (Turning to the boys) My lads, go through the forest southward, till you come to the river. You may then return. My lads, you must not speak the one to the other until I have again seen you. I must have your word on that. Do you promise?BOYS:Sire, we promise.[The Captain leads the boys from the tent.]ACT II, SCENE I: Inside THE EMPEROR'S tent, a few hours later. The CAPTAIN enters with the boys.EMPEROR:Bring up the first boy.[The Captain brings up the MERCHANT'S SON.]EMPEROR:What did you see in the forest?MERCHANT'S SON:Trees, sire.EMPEROR:You saw nothing but trees?MERCHANT'S SON:Just trees.EMPEROR:You may go.MERCHANT'S WIFE:Oh, your Majesty, if you could only see him dance!EMPEROR:Do I want a dancing guide? Captain, bring up the next one.[The Captain brings the MAYOR'S SON.]EMPEROR:What did you see in the forest?MAYOR'S SON:I saw trees and bushes, sire.EMPEROR:Nothing more?MAYOR'S SON:No, sire.EMPEROR:You may go.MAYOR'S WIFE:Oh, your Majesty, if you could only see him ride!EMPEROR:Captain, the last boy.[The Captain brings in PIERRE].EMPEROR:What did you see in the forest?PIERRE:I saw that a man had passed southward just before us, sire.EMPEROR:How did you know that?PIERRE:I saw his footprints.EMPEROR:Did you trace this man by his footprints?PIERRE:Yes, sire, to the river. There were traces of them in the grass and in the mud.EMPEROR:Good! You followed him only to the river.PIERRE:Those were the orders, sire.EMPEROR:The test is over. (To Pierre's mother) Madam, your son shall be my guide. I am proud to have a boy of such keen sight and quick thought in my kingdom. And it is much to be the mother of such a lad. With greatest respect I salute you![He bows to the happy woman with great courtesy.] Which product will result in a sum or difference of cubes The ratio x:y is 3: 1 Which of the following statements is correct? A x is 3/4 of y B y is 1/3 of x C x is 1/3 of y D y is 1/4 of x Part of New York City is located on the Hudson River. The Hudson feeds into one of the world's largest natural harbors. New York City's favorable location has allowed it to become Precise and forceful language State two characteristics of matter demonstrated by: (a)-diffusion (b)-brownian motion 4) DeAndre owns 40 shares of a common stock in a automotive company. Last month the price of the stock was $22.50 per share. Today, the price of the stock is $31.25. By how much did the value of the stock increase? Enter your answers as a number like 105. Which details show that this passage is functioning as an introduction? Check all that apply I really need help with this. Find the equivalent exponential expression.[(-4)^3]^2 PLEASE HELP ASAP! WILL MARK BRAINLIEST In act III, what is Juliets first reaction when she hears that Romeo has killed Tybalt? a) she breaks down and feels sorry for herself b) she tries to think of ways to protect Romeo c) she argues with the nurse about what has happened How much is 1 mole of water molecules? What happens when voters are out of state on Election Dav? Insulin is a protein that is used by the body to regulate both carbohydrate and fat metabolism. A bottle contains 225 mL of insulin at a concentration of 30.0 mg/mL . What is the total mass of insulin in the bottle? According to a survey done at your school, about 42% of all the female students participate in 2 sport seasons. You randomly ask 5 female students at your school how many sport seasons they participate in. What is the probability that at least 3 of the 5 female students surveyed participate in 2 sport seasons? Show your work. are the values of the expressions 2m13 and m+3 equal? 1. Ecology is the study of interactions among organisms. Proteins synthesis is accomplished primarily by the interaction of which two cell structures? A:vacuoles and mitochondria B:ribosomes and vacuoles C:nuclei and ribosomes D:nuclei and mitochondria Please answer correctly !!!! Will mark brainliest !!!!!!!!!!!!!! What is the poem New Scholar By Louise Bennett Coverley about?